D looks more appropriate than E.

In  parts  of  South  America,  vitamin-A  deficiency  is  a  serious  health  problem,  especially  among children. In one region, agriculturists hope to improve nutrition by encouraging farmers to plant a new variety of sweet potato called SPK004 that is rich in beta-carotene, which the body converts into vitamin A. The plan has good chances of success, since sweet potato is a staple of the region’s diet and agriculture, and the varieties currently grown contain little beta-carotene.

Which of the following, if true, most strongly supports the prediction that the plan will succeed?

  1. There are other vegetables currently grown in the region that contain more beta-carotene than the currently cultivated varieties of sweet potato do.
  2. The flesh of SPK004 differs from that of the currently cultivated sweet potatoes in colors and textures, so traditional foods would look somewhat different when prepared from SPK004.
  3. For successful cultivation of SPK004, a soil significantly richer in nitrogen is needed than is needed for the varieties of sweet potato currently cultivated in the region.
  4. There are no other varieties of sweet potato that are significantly richer in beta-carotene than SPK004 is.
  5. The currently cultivated varieties of sweet potato contain no important nutrients that SPK004 lacks.
Expert Asked on June 17, 2017 in Critical Reasoning.
Add Comment
1 Answer(s)

Okay. But are they available in the region? Does it lack in other nutrients?

More importantly, this answer option is out of scope. As the nutrient content in other varieties is not relevant to the nutrient content in this variety. ONLY IF someone places this brand of sweet potato in direct competition with the one being promoted, can we infer that it impacts the sales. Since this is an additional assumption, this answer option is not relevant to evaluate the success of some other potato type. (I cannot use the same logic with E since we are told about varieties that already exist in the region).

If the answer to any of these questions is no, then this option will not strengthen.

Expert Answered on June 18, 2017.
Add Comment

Your Answer

By posting your answer, you agree to the privacy policy and terms of service.